Difference between revisions of "2018 AMC 10A Problems/Problem 22"

(See Also)
Line 20: Line 20:
 
{{AMC10 box|year=2018|ab=A|num-b=21|num-a=23}}
 
{{AMC10 box|year=2018|ab=A|num-b=21|num-a=23}}
  
[[Category:Introductory Number Theory Problems]]
+
[[Category:Intermediate Number Theory Problems]]
 
{{MAA Notice}}
 
{{MAA Notice}}

Revision as of 16:43, 18 June 2018

Let $a, b, c,$ and $d$ be positive integers such that $\gcd(a, b)=24$, $\gcd(b, c)=36$, $\gcd(c, d)=54$, and $70<\gcd(d, a)<100$. Which of the following must be a divisor of $a$?

$\textbf{(A)} \text{ 5} \qquad \textbf{(B)} \text{ 7} \qquad \textbf{(C)} \text{ 11} \qquad \textbf{(D)} \text{ 13} \qquad \textbf{(E)} \text{ 17}$

Solution 1

We can say that $a$ and $b$ 'have' $2^3 * 3$, that $b$ and $c$ have $2^2 * 3^2$, and that $c$ and $d$ have $3^3 * 2$. Combining $1$ and $2$ yields $b$ has (at a minimum) $2^3 * 3^2$, and thus $a$ has $2^3 * 3$ (and no more powers of $3$ because otherwise $gcd(a,b)$ would be different). In addition, $c$ has $3^3 * 2^2$, and thus $d$ has $3^3 * 2$ (similar to $a$, we see that $d$ cannot have any other powers of $2$). We now assume the simplest scenario, where $a = 2^3 * 3$ and $d = 3^3 * 2$. According to this base case, we have $gcd(a, d) = 2 * 3 = 6$. We want an extra factor between the two such that this number is between $70$ and $100$, and this new factor cannot be divisible by $2$ or $3$. Checking through, we see that $6 * 13$ is the only one that works. Therefore the answer is $\boxed{\textbf{(D) } 13}$

Solution by JohnHankock

Solution 2 (Better notation)

First off, note that $24$, $36$, and $54$ are all of the form $2^x\times3^y$. The prime factorizations are $2^3\times 3^1$, $2^2\times 3^2$ and $2^1\times 3^3$, respectively. Now, let $a_2$ and $a_3$ be the number of times $2$ and $3$ go into $a$,respectively. Define $b_2$, $b_3$, $c_2$, and $c_3$ similiarly. Now, translate the $lcm$s into the following: \[\min(a_2,b_2)=3\] \[\min(a_3,b_3)=1\] \[\min(b_2,c_2)=2\] \[\min(b_3,c_3)=2\] \[\min(a_2,c_2)=1\] \[\min(a_3,c_3)=3\] .

(Unfinished) ~Rowechen Zhong

See Also

2018 AMC 10A (ProblemsAnswer KeyResources)
Preceded by
Problem 21
Followed by
Problem 23
1 2 3 4 5 6 7 8 9 10 11 12 13 14 15 16 17 18 19 20 21 22 23 24 25
All AMC 10 Problems and Solutions

The problems on this page are copyrighted by the Mathematical Association of America's American Mathematics Competitions. AMC logo.png